Schwerkraft in anderen als 3 räumlichen Dimensionen und stabile Umlaufbahnen

Ich habe von hier gehört, dass stabile Umlaufbahnen (solche, die eine große Kraft erfordern, um sie deutlich aus ihrer elliptischen Bahn herauszudrücken) nur in drei räumlichen Dimensionen existieren können, da die Schwerkraft in einem zwei- oder vierdimensionalen Raum anders funktionieren würde. Warum ist das?

Nebenbei bemerkt: Wenn Sie über die Verallgemeinerung der Allgemeinen Relativitätstheorie (anstelle der Newtonschen Gravitation) auf verschiedene Dimensionen sprechen, dann unterscheidet sich 2-D tatsächlich stark von 3-D. Insbesondere ist die Raumzeitkrümmung im Vakuum genau null.

Antworten (4)

Konkret bezieht sich das auf das „ Gesetz des umgekehrten Quadrats “, die Natur der Gravitationskraft, dh die Schwerkraft ist umgekehrt proportional zum Quadrat der Entfernung:

F g 1 d 2 .

Wenn Sie dieses Konzept auf das der allgemeinen Kräfte des Potenzgesetzes erweitern (z. B. wenn Sie über den Virialsatz nachdenken ), können Sie schreiben:

F d a ,

Stabile Bahnen sind nur für wenige, spezielle Werte des Exponenten ' möglich a '---insbesondere und genauer gesagt 'geschlossene 1 ', stabile Bahnen treten nur für auf a = 2 (das Abstandsquadratgesetz) und a = 1 ( Hookesches Gesetz ). Dies wird „ Bertrands Theorem “ genannt.

Was hat das nun mit räumlichen Dimensionen zu tun? Nun, es stellt sich heraus, dass bei einer genaueren Beschreibung der Schwerkraft (insbesondere der allgemeinen Relativitätstheorie ) der Exponent des Potenzgesetzes um eins kleiner ist als die Dimension des Raums. Wenn der Raum zum Beispiel zweidimensional wäre, würde die Kraft so aussehen F 1 d , und es gäbe keine geschlossenen Umlaufbahnen.

Beachten Sie auch das a < 3 (und damit 4 oder mehr räumliche Dimensionen) ist bedingungslos instabil, wie in der Antwort von @ nervxxx unten angegeben.


1: Eine „geschlossene“ Umlaufbahn ist eine Umlaufbahn, bei der das Teilchen zu seiner vorherigen Position im Phasenraum zurückkehrt (dh seine Umlaufbahn wiederholt sich).

Sie brauchen keine allgemeine Relativitätstheorie, Sie brauchen nur das Gaußsche Gesetz und den Satz von Stoke, um die d-1-Regel abzuleiten.
@JerrySchirmer danke, guter Punkt --- aber ist der Satz von Stoke nicht erforderlich, wenn die Kraft als Divergenz eines Feldes ausgedrückt wird - das selbst für a = -2 und 1 einzigartig ist?
+1. Also, das ist eigentlich eine Frage 2 + 1 richtig, nicht 1 + 1 ?
Das Problem ist, dass es damit immer noch stabile Umlaufbahnen geben könnte. Die Anfangsgeschwindigkeit müsste einfach gleich rad(Gm) statt rad(Gm/r) sein. Die erforderliche Umlaufgeschwindigkeit wäre nur entfernungsunabhängig. Ich nehme an, das einzige ist, dass elliptische Umlaufbahnen unmöglich wären, worüber wahrscheinlich das Video sprach, das ich verlinkt habe.
@kηives, es kommt explizit auf die räumlichen Dimensionen an, aber das Gesamtergebnis gilt für beliebig viele räumliche Dimensionen.
@zhermes: Die Regel a = {-2,1} stammt aus der Stabilitätsanalyse von Umlaufbahnen. Meines Wissens gibt es keine Grundregel für die Ableitung, wie Sie in Ihrem letzten Absatz andeuten. Das würde ich erwarten, dass es nicht von der Dimensionalität des Raums beeinflusst wird (solange D 2 ), da die Instabilität von radialen Störungen herrührt, die nicht durch das Hinzufügen oder Entfernen von Dimensionen beeinflusst werden. Fast jede Analyse sagt, dass da etwas Besonderes ist 3 + 1 Maße.
@zhermes Während alles, was Sie über geschlossene Umlaufbahnen sagen, richtig ist, gibt es tatsächlich ein echtes Gefühl der Instabilität, das in 4+ Dimensionen einsetzt: Jede Störung einer Umlaufbahn sendet die Trennung entweder ins Unendliche oder auf 0. Siehe die Antwort von nervxxx, die im Grunde genommen folgt der Methode in zB Goldstein.
@JerrySchirmer, ich stimme voll und ganz zu, ich habe angesprochen, wie Sie das besondere Potenzgesetz der Schwerkraft ableiten, nicht die Potenzgesetze der Stabilität.
@zhermes Sie leiten das besondere Potenzgesetz der Schwerkraft ab N räumliche Dimensionen durch Auffinden der zugehörigen Green-Funktion der PDE. Im Allgemeinen z N 2 , das ist F = r / S N 1 | r | N , wo S k ist die Oberfläche der k -Kugel. Daraus können Sie das zugehörige Potenzial für beliebig viele Dimensionen entnehmen.
@zhermes: und das besondere Potenzgesetz der Schwerkraft stammt aus dem Satz von Stoke.
Diese Antwort ist mathematisch korrekt, aber ich denke, die Übersetzung von Mathematik ins Englische ist falsch und beantwortet die ursprüngliche Frage nicht. Wir sprechen von Stabilität, nicht von Schließung. Andere haben gesagt, dass Sie stabile Umlaufbahnen im 2-D-Raum haben können. Ich füge hinzu, dass selbst in unserem 3-D-Raum elliptische Umlaufbahnen nicht geschlossen sind, sie präzedieren gemäß der allgemeinen Relativitätstheorie. (Und wenn es darauf ankommt, sind sie auch nicht perfekt stabil.)
@SpacelikeCadet, wenn es darauf ankommt, ist nichts stabil.
@zhermes Vielleicht hätte ich das nicht erwähnen sollen ... Auf jeden Fall sind Umlaufbahnen in einigen Dimensionen viel stabiler als in anderen, und das Schließen ist nebensächlich.
@RetardedPotential, ja, ich denke, wir sind uns einig. Das a > -2 ist das stärkste Stabilitätskriterium.
Ich frage mich, was passiert, wenn man einfach ein anderes Potenzial nutzt, das man noch hat F 1 / r 2 auch in höheren Dimensionen?
@Garan Warum denkst du, dass das eine stabile Umlaufbahn beschreibt? Wenn die Umlaufgeschwindigkeit unabhängig von der Höhe ist, was passiert mit einem umlaufenden Objekt, das gestört wird?

Ich werde versuchen, es zu beantworten, indem ich radiale Abweichungen von einer kreisförmigen Umlaufbahn betrachte. Zunächst müssen wir zwei Dinge über unser n-dimensionales Universum annehmen: Das zweite Newtonsche Gesetz gilt immer noch, das heißt,

für den Positionsvektor eines Partikels in n-Dimensionen x = ( x 1 , x 2 , x n ) ,

m x ¨ = F ,
wo F ist eine n-dimensionale Kraft,

und auch, dass das Gravitationsgesetz durch das Gaußsche Gesetz gegeben ist:

g = 4 π G ρ ,
wo g ist das Gravitationskraftfeld. (Siehe Wikipedia für weitere Informationen).

Die Lösung für diese pde ist

g ∼= r 1 n e r ^ ,
Pro n 2 . (Für n = 1 die Bewegung ist auf einer Linie und weil es immer attraktiv ist, wird die 'Umlaufbahn' immer noch eine 'Umlaufbahn' bleiben)

Da die Bewegung immer darauf beschränkt ist, sich in der 2-Ebene zu bewegen, die durch den anfänglichen radialen Vektor aufgespannt wird r 0 und der Anfangsgeschwindigkeitsvektor v 0 , ist es am einfachsten, die Bewegung in Zylinderkoordinaten zu analysieren. Das heißt, Newtons zweites Gesetz wird

m ( r ¨ θ ˙ 2 r ) = F r m ( r θ ¨ + 2 r ˙ θ ˙ ) = F θ m x 3 ¨ = F x 3 m x 4 ¨ = F x 4 m x n ¨ = F x n ,
wo x 1 und x 2 sind Koordinaten der aufgespannten Ebene v 0 und r 0 . Hier r wirklich bedeutet x 1 2 + x 2 2 , aber es stellt sich heraus, dass die Bewegung nur 2-D ist, dh x 3 = x 4 = x n = 0 , Wir können sagen r = x 1 2 + + x n 2 .

Jetzt machen wir uns die Tatsache zunutze, dass die Schwerkraft immer radial ist, also F θ = 0 und wir können die ersten beiden Gleichungen kombinieren, um zu erhalten

r ¨ L 2 r 3 = F r = f ( r ) ,
wo L ist eine Bewegungskonstante (in 3D ist dies der Drehimpuls).

Für eine Kreisbahn bei r = r c , r ¨ = 0 , also bleiben wir übrig

L 2 r 3 = f ( r ) .
Betrachten Sie kleine Abweichungen von r c : x = r r c . Wenn man dies in das Newtonsche Gesetz einfügt und zur ersten Ordnung erweitert, erhält man
x ¨ + [ 3 f ( r c ) / r c f ' ( r c ) ] x = 0.
Dies ist eine einfache harmonische Gleichung, wenn das Zeug in der Klammer positiv ist. Wir erhalten also eine Stabilitätsbedingung
[ 3 f ( r c ) / r c f ' ( r c ) ] > 0.

Prüfen wir dies anhand einer Radialkraft f ( r ) = k r d . Die Stabilitätsbedingung gibt

k r c d k d 3 r c d < 0 ,
was impliziert d > 3 . Also wenn das Kraftgesetz so geht r d wo d > 3 , dann ist die Umlaufbahn nicht stabil. Das kann man mit etwas mehr Arbeit zeigen d = 3 ist auch instabil.

Also zu den Maßen n 4 , die Umlaufbahn ist instabil. Es scheint jedoch, dass z d = 1 oder 2 , die Umlaufbahn ist stabil, was uns zu dem Ergebnis führt, dass Umlaufbahnen in 3 Dimensionen (unsere Welt) und auch die in 2 Dimensionen stabil sind, was der Aussage des Videos widerspricht. Ich könnte mich aber irren.

Beifall.

Einverstanden. (Außer ich glaube, du wolltest sagen d < 3 bedeutet nicht stabil.) n 4 ist instabil, während n = 2 hat keine geschlossenen Umlaufbahnen, ist aber stabil.

Ein kurzer Punkt, der zu den oben geposteten Antworten hinzugefügt werden kann, obwohl ich nicht so tun kann, als würde ich die ganze Mathematik verstehen:

Soweit ich weiß, sind Umlaufbahnen in 2D in dem Sinne stabil, dass der umlaufende Körper nicht entkommt oder in den Primärkörper kollabiert. Siehe zB

https://www.reddit.com/r/askscience/comments/q8fmo/what_would_orbits_look_like_in_a_2d_universe/

Wenn die 2D-Kraft auf 1/r abfällt, ist ihr Potential logarithmisch – was bedeutet, dass die Fluchtgeschwindigkeit unendlich ist. Dies ist ziemlich einfach zu zeigen; sogar ich kann das.

Wenn ich das richtig verstanden habe, scheinen die Augenhöhlen aber meistens nicht geschlossen, sondern wie Blütenblätter geformt zu sein. Für nahezu kreisförmige Umlaufbahnen wäre das nicht unbedingt ein allzu großes Problem.

Ich nehme an, dies spricht über die Newtonsche Schwerkraft (dh nicht über die Relativitätstheorie). Betrachten wir das effektive Potenzial:

v eff ( r ) = L 2 2 m r 2 + v ( r )

wo v ist die gewöhnliche potentielle Energie, und L ist der Drehimpuls. Zunächst mag man fragen, warum das effektive Potential diese Form hat. Denken Sie daran, dass für ein einzelnes Teilchen L = m r 2 ω , das ist also äquivalent,

v eff ( r ) = ω 2 r 2 2 m + v ( r )

Dieser erste Term ergibt sich aus den Bewegungsgleichungen für ein freies Teilchen. Es ist zweckmäßig, es in Form des Drehimpulses zu formulieren, da der Drehimpuls unter zentralen Kräften eine Erhaltungsgröße ist.

Warum nutzen wir das effektive Potenzial? Weil es uns hilft, nur über die radialen Bewegungen eines Teilchens zu sprechen und die Winkelbewegungen mit dem realen Potenzial in einen Topf zu werfen. Ein lokales Extremum des effektiven Potentials sagt uns etwas über einen Gleichgewichtsabstand.

Jetzt, in 3D, das Potenzial v ( r ) denn die Schwerkraft ist G M m / r . Was dies bedeutet, ist, dass, wie r 0 , wird das effektive Potential dank des Drehimpulsanteils schließlich explodieren, den Gravitationsanteil überwinden und das Teilchen wieder nach außen zwingen, es sei denn, es liegt auf einer direkten Einfallsbahn.

In 2d ist das Potenzial anders. Warum ist das? Die Newtonsche Gravitation befasst sich mit Differentialgleichungen der Form 2 v ρ . Die Punktquellenlösung dieser Gleichung (die Greensche Funktion) ist proportional zu ln r --vergleichen Sie zum Beispiel das elektrische Potential einer unendlichen Linienladung. Dies ist genau die gleiche Geometrie und Differentialgleichung, zumindest in der Struktur.

Lassen Sie uns für eine Sekunde überprüfen, ob dies der Fall ist. Lassen v = C ln r in 2d für einige konstant C . Dann ist die Gravitationskraft

F = v r = C / r

was innerlich für alles Positive ist C . Das ist wichtig. In 2d sieht unser effektives Potenzial dann so aus,

v eff = K r 2 + C ln r

für zwei Konstanten K , C . Die Kraft ist

F eff = 2 K r 3 C r 1 = r 1 ( 2 K r 2 + C )

So r Gl = 2 K / C . Aber ist dieses Gleichgewicht stabil?

F eff r = 6 K r 4 + C r 2

Bei r Gl , dies wird ausgewertet 6 C 2 / 4 K + C 2 / 2 K = C 2 / K .

Hm. Das würde darauf hindeuten, dass der Gleichgewichtspunkt stabil ist. Also, vielleicht hat jemand eine Referenz, um dies vorzuschlagen. Ich stecke fest.

Geht das effektive Potential zu wie beide r 0 und r , dann muss sie mindestens ein stabiles Minimum haben, also gibt es mindestens eine stabile Kreisbahn. Eine potentielle Energie proportional zu ln r würde dieses Kriterium erfüllen.